Find the measure of PR.
PLEASE HELP ASAP!!
A. 18
B. 19
C. 25
D. 22

Answers

Answer 1

Answer:

A. 18

Step-by-step explanation:

the equation :

RF×RS = PR×RQ

16×9 = PR × 8

144 = PR×8

PR = 144/8

PR = 18

Answer 2

If the length of FR is 16 units then the length of PR is 18 units which is option A.

What is circle?

Circle is a figure whcih is having an arc showing points which are at equidistant from the centre point. The distance of arc from the centre point is known as radius.

How to find length?

From the rule of tangents and point outside the circle we can say that,

RF×RS = PR×RQ

16×9 = PR × 8

144 = PR×8

PR = 144/8

PR = 18 units

Hence if the length of FR is 16 units then the length of PR is 10 units.

Learn more about circle at https://brainly.com/question/24375372

#SPJ2


Related Questions

Si Juana tiene dos perros y lo simbolizamos y 2p y le regalan un hato y lo simbolizamos por g como se representa en el leguaje algebraico.

A) 3P
B) 3G
C) 2PTG
D) 2P-G

Answers

your question makes absolutely no sense

hurry PLEASE i'm unsure of this

Answers

Answer:

Infinite solutions

Step-by-step explanation:

x + 4y = 7 --------------(I)

8y = 14 - 2x

⇒ 2x + 8y = 14 -----------(II)

[tex]\frac{a_{1}}{a_{2}}=\frac{1}{2}\\\\\frac{b_{1}}{b_{2}}=\frac{4}{8}=\frac{1}{2}\\\\\frac{c_{1}}{c_{2}}=\frac{7}{14}=\frac{1}{2}\\\\\frac{a_{1}}{a_{2}}=\frac{b_{1}}{b_{2}}=\frac{c_{1}}{c_{2}}=\frac{1}{2}[/tex]

So, infinite solutions

Answer:

G

Step-by-step explanation:

Given the 2 equations

x + 4y = 7 → (1)

8y = 14 - 2x → (2)

make x the subject of (1) by subtracting 4y from both sides

x = 7 - 4y

Substitute x = 7 - 4y into (2)

8y = 14 - 2(7 - 4y) ← distribute

8y = 14 - 14 + 8y

8y = 8y

Since both sides are equal this indicates the system has infinite solutions

2x²+3x²+3.(-1) =5x.x+5x .1

Answers

[tex]\\ \sf\longmapsto 2x^2+3x^2+3(-1)=5x.x+5x.1[/tex]

[tex]\\ \sf\longmapsto 5x^2-3=5x^2+5x[/tex]

[tex]\\ \sf\longmapsto 5x^2-5x^2-3=5x[/tex]

[tex]\\ \sf\longmapsto 5x=-3[/tex]

[tex]\\ \sf\longmapsto x=\dfrac{-3}{5}[/tex]

I need help please help me

Answers

Answer:

I found x for you, right there.

Answer:  x is 12 and y is 5

Step-by-step explanation:

Find the measure of the indicated angle to the nearest degree.

Answers

Answer:

37

Step-by-step explanation:

tan(theta)=perpendicular/base

tan(theta)=3/4

theta=arctan(3/4)=37

Answer:

37°

Step-by-step explanation:

for this question you have to use the tan ratio since the opposite of that angle has been given as 3 and the adjacent has also been given which is 4..I will represent the unknown angle using x so,

tan x=opposite/adjacent

tanx=3/4

tanx=0.75

x=tan inverse of 0.75

x=36.9 or 37°

I hope this helps

Divide. Write your answer as a fraction in simplest form. − 10 2/7÷(−4 4/11)=

Answers

Answer:

33/14

Step-by-step explanation:

[tex] - 10 \frac{2}{7} + ( - 4 \frac{4}{11} )[/tex]

[tex] = - \frac{72}{7} \div - ( \frac{48}{11} )[/tex]

[tex] = \frac{72}{7} \times \frac{11}{48} [/tex]

[tex] = \frac{3}{7} \times \frac{11}{2} [/tex]

[tex] = \frac{33}{14} [/tex]

stan dreamcatcher

Given: j(x) = x2 - 2x + 1
Which set of values represents the range of the function for the domain {0, 1, 5}?

Answers

Answer:

{1, 0, 16}

Step-by-step explanation:

given..

j(x) = x^2-2x+1

put all given values of domain (1,0 and 5 ) in the equation..

the values you get are range of the function

Answer: 1

Step-by-step explanation

0(2)-2(0)+1=0+0+1= 1

1(1)-(2)(1)+1=1-2+1= 1

(5)(2)-(2)(5)+1=10-10+1= 1

Is the following number rational or irrational?
-117
Choose 1 answer:
Rational
Irrational

Answers

Answer:

-117 is irrational number

Answer:

Irrational

Step-by-step explanation:

Irrational number can't be written as a faction, -11pie can't be written as a fraction. Therefore it is a irrational number.

Find the surface area of the triangular prism

Answers

Front: 12 square centimeters
1/2 x 6 x 4 = 12

Back: 12 square centimeters
1/2 x 6 x 4 = 12

Right: 15 square centimeters
3 x 5 = 15

Left: 15 square centimeters
3 x 5 = 15

TOTAL: 54 square centimeters

The surface area is the added areas of each face of a 3D figure.
Formulas I’ve used:
Area of a parallelogram:
A = bh

Area of a triangle:
A = 1/2bh

Add.
(3x2 – 2x) + (4x-3)
O A. 7x2- 5x
O B. 12x3 - 14x2 + 6x
O C. 3x2 - 6x + 3
O D. 3x2 + 2x-3

Answers

Answer:

O D. 3x2 + 2x-3

Answer:

A.7x2-5x questions 2of 20

Trigonometric ratios
class 9
please answer my questions​

Answers

Step-by-step explanation:

Hi there!

Please see the answer in the picture.

Hope it helps!

1. Approach

One is given a trigonometric equation with and one is asked to prove that it is true. Using the attached image, combined with the knowledge of trigonometry, one can evaluate each trigonometric function. Then one can simplify each ratio to solve. To yield the most accurate result, one has to each of the ratios in a fractional form, rather than simplifying it into a decimal form. Remember the right angle trigonometric ratios, these ratios describe the relationship between the sides and angles in a right triangle. Such ratios are as follows,

[tex]sin(\theta)=\frac{opposite}{hypotenuse}\\\\cos(\theta)=\frac{adjacent}{hypotenuse}\\\\tan(\theta)=\frac{opposite}{adjacent}\\\\csc(\theta)=\frac{hypotenuse}{opposite}\\\\sec(\theta)=\frac{hypotenuse}{adjacent}\\\\cot(\theta)=\frac{adjacent}{opposite}[/tex]

Please note that the terms (opposite) and (adjacent) are relative to the angle uses in the ratio, however the term (hypotenuse) refers to the side opposite the right angle, this side never changes its name. Use these ratios to evaluate the trigonometric functions. Then simplify to prove the identity.

2. Problem (9)

[tex]\frac{sin(60)+cos(30)}{1+sin(30)+cos(60)}=sin(60)[/tex]

As per the attached image, the following statements regarding the value of each ratio can be made:

[tex]sin(60)=\frac{\sqrt{3}}{2}\\\\cos(30)=\frac{\sqrt{3}}{2}\\\\sin(30)=\frac{1}{2}\\\\cos(60)=\frac{1}{2}[/tex]

Substitute,

[tex]\frac{sin(60)+cos(30)}{1+sin(30)+cos(60)}=sin(60)[/tex]

[tex]\frac{\frac{\sqrt{3}}{2}+\frac{\sqrt{3}}{2}}{1+\frac{1}{2}+\frac{1}{2}}=\frac{\sqrt{3}}{2}[/tex]

Simplify,

[tex]\frac{\frac{\sqrt{3}}{2}+\frac{\sqrt{3}}{2}}{1+\frac{1}{2}+\frac{1}{2}}=\frac{\sqrt{3}}{2}[/tex]

[tex]\frac{\frac{2\sqrt{3}}{2}}{1+\frac{1}{2}+\frac{1}{2}}=\frac{\sqrt{3}}{2}[/tex]

[tex]\frac{\frac{2\sqrt{3}}{2}}{1+1}=\frac{\sqrt{3}}{2}[/tex]

[tex]\frac{\sqrt{3}}{1+1}=\frac{\sqrt{3}}{2}[/tex]

[tex]\frac{\sqrt{3}}{2}=\frac{\sqrt{3}}{2}[/tex]

Thus, this equation is true.

2. Problem (10)

Use a similar strategy to evaluate this equation,

[tex]\frac{1-cos(30)}{sin(30)}=\frac{1-cot(60)}{1+cot(60)}[/tex]

Use the attached image to evaluate the ratios.

[tex]cos(30)=\frac{\sqrt{3}}{2}\\\\sin(30)=\frac{1}{2}\\\\cot(60)=\frac{1}{\sqrt{3}}[/tex]

Substitute,

[tex]\frac{1-cos(30)}{sin(30)}=\frac{1-cot(60)}{1+cot(60)}[/tex]

[tex]\frac{1-\frac{\sqrt{3}}{2}}{\frac{1}{2}}=\frac{1-\frac{1}{\sqrt{3}}}{1+\frac{1}{\sqrt{3}}}[/tex]

Simplify,

[tex]\frac{1-\frac{\sqrt{3}}{2}}{\frac{1}{2}}=\frac{1-\frac{1}{\sqrt{3}}}{1+\frac{1}{\sqrt{3}}}[/tex]

[tex]\frac{\frac{2-\sqrt{3}}{2}}{\frac{1}{2}}=\frac{1-\frac{1}{\sqrt{3}}}{1+\frac{1}{\sqrt{3}}}[/tex]

[tex]\frac{\frac{2-\sqrt{3}}{2}}{\frac{1}{2}}=\frac{\frac{\sqrt{3}-1}{\sqrt{3}}}{1+\frac{1}{\sqrt{3}}}[/tex]

[tex]\frac{\frac{2-\sqrt{3}}{2}}{\frac{1}{2}}=\frac{\frac{\sqrt{3}-1}{\sqrt{3}}}{\frac{\sqrt{3}+1}{\sqrt{3}}}[/tex]

[tex]2-\sqrt{3}=\frac{\frac{\sqrt{3}-1}{\sqrt{3}}}{\frac{\sqrt{3}+1}{\sqrt{3}}}[/tex]

[tex]2-\sqrt{3}=\frac{\sqrt{3}-1}{\sqrt{3}}*\frac{\sqrt{3}}{\sqrt{3}+1}}[/tex]

[tex]2-\sqrt{3}=\frac{\sqrt{3}-1}{\sqrt{3}+1}[/tex]

Rationalize the denominator,

[tex]2-\sqrt{3}=\frac{\sqrt{3}-1}{\sqrt{3}+1}[/tex]

[tex]2-\sqrt{3}=\frac{\sqrt{3}-1}{\sqrt{3}+1}*\frac{\sqrt{3}-1}{\sqrt{3}-1}[/tex]

[tex]2-\sqrt{3}=\frac{(\sqrt{3}-1)^2}{3-1}[/tex]

[tex]2-\sqrt{3}=\frac{3-2\sqrt{3}+1}{2}[/tex]

[tex]2-\sqrt{3}=\frac{4-2\sqrt{3}}{2}[/tex]

[tex]2-\sqrt{3}=2-\sqrt{3}[/tex]

Therefore, this equation is also true.

u4gent help needed
help me with the question of o.math​

Answers

Answer:

1≤f(x)≤5

Step-by-step explanation:

-1≤x≤1

-2≤2x≤2 (Multiplied by 2 both side)

-2+3≤2x+3≤2+3 (Adding three both sides)

1≤f(x)≤5

Find the first three terms of the sequence given by the following.
a
n = 25-3(n − 1), n= 1, 2, 3, ...

A. 28, 25, 22
B. 25, 22, 19
C. 25, 28, 31
D. 28, 31, 34

Answers

the answer is

A. 28, 25, 22

HELP ITS DUE IN THE MORNING AND ITS 3:57​

Answers

Answer:

A " (1,-2)

B " (4,0)

C " (6,-3)

Step-by-step explanation:

Hope it helped.

° ° °

a sum of money Doubles itself in 5 years what is rate of simple interest​

Answers

Step-by-step explanationIf you are reading this say

thank u


Which segment is parallel to ED?

Answers

Answer:

AB

Step-by-step explanation:

The segments that are parallel need to be in the same direction ( up and down)

The segments that are parallel are FH, AB, GC

Answer:

AB

Step-by-step explanation:

since is a cube all of the angles are 90 degees and this only possibel whn the line That a vertical a parrelllt to each other

I need help ASAP!!! Please help me FInd the missing side

Answers

Answer:

Step-by-step explanation:

Tan 22 = [tex]\frac{opposite \ side}{adjacent \ side}[/tex]

[tex]tan \ 22 = \frac{30}{x}\\\\0.4040 = \frac{30}{x}\\\\0.4040*x = 30\\\\x = \frac{30}{0.4040}\\\\x = 74.25\\\\x = 74.3[/tex]

Answer:

Sin= opp/hyp

Sin22= 30/x

0.3746=30/x

cross multiply

0.3746x=30

make x the subject of the formula

x= 30/0.3746

x=80

The circumference of a circle is 20π. What is the area of the circle?

Answers

Answer:

The area of the circle is 100 square units.

Step-by-step explanation:

We are given that the circumference of a circle is 20π, and we want to determine its area.

Recall that the circumference of a circle is given by the formula:

[tex]\displaystyle C = 2\pi r[/tex]

Substitute:

[tex]20 \pi = 2 \pi r[/tex]

Solve for the radius:

[tex]\displaystyle r = \frac{20\pi}{2\pi} = 10[/tex]

The area of a circle is given by:

[tex]\displaystyle A = \pi r^2[/tex]

Since the radius is 10 units:

[tex]\displaystyle A = \pi (10)^2[/tex]

Evaluate:

[tex]\displaystyle A = 100\pi\text{ units}^2[/tex]

In conclusion, the area of the circle is 100 square units.

Can some help please

Answers

Answer:

Step-by-step explanation:

2 * 10^7   You are to use a single digit. That's the 2 on the left. Then count what it takes to get the decimal between the 2 and the 3. It's 7

0.000136

Count the number of zeros. Add a minus 1. You want the number to be counted until you get minus 1 which is the number of powers after the 1.

1 * 10^-4

26837 becomes 2 * 10^4. 4 is the number of digits you have before you get to a number between 1 and 10.

0.0302 becomes 3 * 10^-(1 + 1) = 3 * 10^-2

round off to one decimal place please ​

Answers

Answer:

Theta = 37.9 degrees

AC = 11.4

Step-by-step explanation:

Since this is a right triangle, we can use trig functions

tan theta = opp/ adj

tan theta = 7/9

tan ^ -1 tan theta = tan ^-1 (7/9)

theta =37.87498

To 1 decimal place

Theta = 37.9 degrees

We use the Pythagorean theorem to find AC

a^2 + b^2 = c^2

7^2 + 9^2 = AC^2

49+81 = AC^2

130 = AC^2

Taking the square root of each side

sqrt(130) = sqrt(AC^2)

AC = 11.40175

Rounding to 1 decimal place

AC = 11.4

How many gallons equal 26 liters

Answers

Answer:

6.8 gallions i believe. im not quite sure

helpppppppppppppp meeeeeeeeeeeeeeeee plsssssssssssssssss!!!!!!!!!!!!!

Answers

Answer:

a=13

Step-by-step explanation:

-90=-6a-12

-6a=-78

a=13

Answer:

a=13 well as u can see the other comment already gave an explanation of y it is a=13 and i completely agree with it have a nice afternoon,night,or day to u

Step-by-step explanation:

please help i need to finnish this!

Answers

Answer:

56

Step-by-step explanation:

f(x)=6*x^2+2 =6*3^2+2=56

Answer:

56

Step-by-step explanation:

6(x^2) + 2  = 0replace x =  3 => 6 x (3 ^ 2) + 2 = 56

I don't understand need help?

Answers

9514 1404 393

Answer:

  2. (only)

Step-by-step explanation:

The Pythagorean theorem tells you the sum of the squares of the two legs of a right triangle is equal to the square of the hypotenuse. To determine if these are right triangles, determine if that condition is met.

  1. 3^2 +5^2 = 9 + 25 = 34 ≠ (√35)^2 . . . . not a right triangle

  2. 5^2 +4^2 = 25 +16 = 41 = (√41)^2 . . . a right triangle

  3. 6^2 +8^2 = 36 +64 = 100 ≠ (√10)^2 . . . . not a right triangle

  4. 3^2 +3^2 = 9 +9 = 18 ≠ (3√3)^2 = 27 . . . . not a right triangle

find the mean value of the following. 5, 11, 4, 10, 8, 6​

Answers

5+11+4+10+8+6 = 44
44/6 = 7.33….


answer: 7.33…..

If 75% is 10 days, how many days is the remaining 25%?

Answers

Answer:

10/3 days  or 3 1/3 days

Step-by-step explanation:

Let d = number of days total

75% of the days is 10

.75 d = 10

d = 10/.75

d = 40/3 days

We want to know how man is 25% of d

d * 25%

(40/3) *.25

10/3 days

Am I suppose to substitute the variables with random numbers in order to answer these questions???

Translation A maps (x, y) to (x + n, y + 1). Translation B maps (x, y) to (x +s, y + m).
1. Translate a point using Translation A, followed by Translation B. Write an algebraic rule for the final image of the point after this composition.
2. Translate a point using Translation B, followed by Translation A. Write an algebraic rule for the final image of the point after this composition.
3. Compare the rules you wrote for parts (a) and (b). Does it matter which translation you do first? Explain your reasoning.

Answers

9514 1404 393

Answer:

(x, y) ⇒ (x +n +s, y +1 +m)(x, y) ⇒ (x +s +n, y +m +1)they are identical in effect; order does not matter

Step-by-step explanation:

Substitute the expressions.

A then B

After the first translation, the value of x is (x+n). Put that as the value of x in the second translation.

  x ⇒ x +s . . . . . . . . . the definition of the second translation

  (x+n) ⇒ (x+n) +s . . . the result after both translations

The same thing goes for y. After the first translation, its new value is (y+1).

  y ⇒ y +m . . . . . . . . the definition of the second translation

  (y+1) ⇒ (y+1) +m . . . the result of both translations

Then the composition of A followed by B is (x, y) ⇒ (x +n +s, y +1 +m).

__

B then A

The same reasoning applies. After the B translation, the x-coordinate is (x+s) and the y-coordinate is (y+m). Then the A translation changes these to ...

  x ⇒ x +n . . . . . . . . . . definition of translation A

  (x+s) ⇒ (x+s) +n . . . . translation A operating on point translated by B

  y ⇒ y +1 . . . . . . . . . . . definition of translation A

  (y+m) ⇒ (y+m) +1 . . . . translation A operating on point translated by B

The composition of B followed by A is (x, y) ⇒ (x + s + n, y + m + 1).

__

You should recognize that the sums (x+n+s) and (x+s+n) are identical. The commutative and associative properties of addition let us rearrange the order of the terms with no effect on the outcome.

The two translations give the same result in either order.

solve for w.
-9/7=-2/3w-1/2

Answers

Answer: [tex]w=\frac{33}{28}[/tex]

Step-by-step explanation:

To solve for w, we want to isolate w.

[tex]-\frac{9}{7}=-\frac{2}{3}w-\frac{1}{2}[/tex]            [add both sides by 1/2]

[tex]-\frac{11}{14}=-\frac{2}{3}w[/tex]                 [multiply both sides by -3/2]

[tex]w=\frac{33}{28}[/tex]

Now we know that [tex]w=\frac{33}{28}[/tex].

Answer:

[tex]\sf w=\dfrac{33}{28} \\[/tex]

Step-by-step explanation:

[tex]\sf -\dfrac{9}{7} =-\dfrac{2w}{3} -\dfrac{1}{2}[/tex]

First, take -2w/3 to the left side.

[tex]\sf -\dfrac{9}{7}+\dfrac{2w}{3} = -\dfrac{1}{2}[/tex]

Then, add 9/7 to both sides.

[tex]\sf \dfrac{2w}{3} = -\dfrac{1}{2}+\dfrac{9}{7}[/tex]

Make the denominators the same and add the fractions.

[tex]\sf \dfrac{2w}{3} = -\dfrac{1*7}{2*7}+\dfrac{9*2}{7*2}\\\\\sf \dfrac{2w}{3} = -\dfrac{7}{14}+\dfrac{18}{14}\\\\\sf \dfrac{2w}{3} = \dfrac{-7+18}{14}\\\\\sf \dfrac{2w}{3} = \dfrac{11}{14}[/tex]

Use cross multiplication.

[tex]\sf 2w*14=11*3\\\\28w=33[/tex]

Divide both sides by 28.

[tex]\sf w=\dfrac{33}{28} \\[/tex]

please help me guys its important ​

Answers

Answer:

1441.08in^3

Step-by-step explanation:

An angle measures 73.6° more than the measure of its complementary angle. What is the measure of each angle?

PLEASE help, I'm struggling a lot!

Answers

Answer:

Let ABC = 73.6

Complement = ABD = 16.4

ABx = unknown angle

ABx + (ABx + 73.6) = 90

ABx = 16.4 / 2 = 8.2

The angles are 8.2 and (8.2 + 73.6) = 90

Other Questions
find the slope of the tangent line of the curve r = cos (3theta) at theta = pi / 3 Factorize :solve no g and h maths class 9Multiply: 412 212 Nahuatl is the language spoken by the Nahua ethnic group that is found today in Mexico, but with deep historical roots. You might know one Nahua group: the__________________, more accurately called the Mexica. The Mexica were one of many Mesoamerican cultural groups that flourished in Mexico prior to the arrival of Europeans in the sixteenth century. please help me with this this is Computer chapter Advanced HTML of class 8th what are the phages of development ? Select the Correct Answer.Which Question would MOST LIKELY indicate that a passage has a problem-and-solution text structure?/*answer key in photo* Enter the ratio as a fraction in lowest terms6 minutes to 30 minutes. ****URGENT****I need help!!!the answers are there, i need someone to explain please! What are the students observations and inferences before he starts his investigation? For those who eat too much McDonalds, do you feel it is right to sue them? One reason why you feel that way. Cho f l hm chn, g l hm l. Tnh gi tr ca (gf)(4,7), bit g(5,9)=7,9 v f(4,7)=5,9. HELP!!Consider the polynomial solve for x please help (show work) Dental plaque Group of answer choices anchors teeth to their bony sockets. is calcified organic matter on the surface of teeth. protects teeth from bacteria-induced tooth decay. forms a bone-like protective layer. consists of food particles trapped in a sticky matrix. What are the effects of global warming? Find the length of the other two sides isosceles right triangle What is 98.907 rounded to 1 significant figure? Deion is saving up to buy a new phone. He already has $95 and can save an additional $7 per week using money from his after school job. How much total money would Deion have after 6 weeks of saving? Also, write an expression that represents the amount of money Deion would have saved in w weeks. A study was conducted to determine whether magnets were effective in treating pain. The values represent measurements of pain using the visual analog scale. Assume that both samples are independent simple random samples from populations having normal distributions. Use a 0.05 significance level to test the claim that those given a sham treatment have pain reductions that vary more than the pain reductions for those treated with magnets.Sham n= 20 x=0.41 s=1.37Magnet n= 20 x =0.46 s= 0.94Identify the test statistic. F=Identify P-Value=What is the conclution for the hypothesis test?A. Fail to reject the null hypothesis. There is insufficient evidence to to support the claim that those given a sham treatment have reductions that vary more than those treated with magnetsB. Reject the null hypothesis. There is insufficient evidence to to support the claim that those given a sham treatment have reductions that vary more than those treated with magnetsC.Fail to reject the null hypothesis. There is sufficient evidence to to support the claim that those given a sham treatment have reductions that vary more than those treated with magnetsD.Reject the null hypothesis. There is sufficient evidence to to support the claim that those given a sham treatment have reductions that vary more than those treated with magnets